Wie kann man für eine Schleife, die sich in einem konstanten Magnetfeld bewegt, ε=IRε=IR\varepsilon=IR schreiben?

Eine Schleife bewegt sich nach unten, wobei ein Teil davon in einem konstanten Magnetfeld wie gezeigt in den Bildschirm zeigt. Wir wissen, dass die EMK aufgrund der Magnetkraft gegeben ist durch

E = D Φ D T .

Wenn die Schleife einen Gesamtwiderstand von R Warum wird dann der Strom in der Schleife sein?

ICH = E / R .Geben Sie hier die Bildbeschreibung ein

Was wäre es sonst?
Ohmsches Gesetz ist ICH = v R wir haben nicht v hier ist es stattdessen e, das sich von V unterscheidet
Die EMF ε = D Φ D T liefert die Potentialdifferenz zwischen zwei Punkten auf der Schleife, wenn sie durch die fällt B -Feld.
@Physics et al., wie?
Ich schlage vor, dass Sie sich über Faradays Gesetz und Bewegungs-EMK informieren.
Diese Gleichung wird von Griffiths verwendet, bevor er in Problem 7.11 zum Faradayschen Gesetz übergeht, also bin ich sicher, dass die Antwort nicht das Faradaysche Gesetz ist.
Okay, gut, ich schaue später bei Griffith vorbei, da ich etwas beschäftigt bin. Wie Jacob1729 betonte, kann es seit der Gegen-EMK nichts anderes als das Ohmsche Gesetz sein. ε , wird durch den magnetischen Fluss durch die Schleife erzeugt, der sich mit der Zeit ändert, wenn die Schleife fällt. Dies induziert gemäß dem Lenzschen Gesetz einen Strom in der Schleife. Es kann also eigentlich nichts anderes sein als ε = ICH R .
Lesen Sie die Abschnitte 7.1.2 und 7.1.3. Vielleicht lesen Sie den gesamten Abschnitt 7.1.
Ich habe sie immer und immer wieder gelesen, ich wäre Ihnen dankbar, wenn Sie auf den Beweis hinweisen könnten.
Sie werden KEINEN Beweis dafür finden ich = E / R in jedem Lehrbuch, das sich auf in einer geschlossenen Leiterschleife induzierten Strom bezieht, da diese Formel nicht die korrekten Ströme für alle Werte von R berechnet. Diese Formel ignoriert den entgegengesetzten magnetischen Fluss, der durch den in der Schleife induzierten Strom erzeugt wird, und ergibt einen absurden UNENDLICHEN Strom für jede Nicht-Null E Wenn R = 0 . Es gibt auch fehlerhafte Ströme für kleine Widerstände ungleich Null.

Antworten (5)

Gute Frage. Meiner Erfahrung nach geben die meisten Einführungen in E&M keine großartige Erklärung der genauen Ähnlichkeiten und Unterschiede zwischen Spannung und elektromotorischer Kraft (EMK) und wann Sie die Konzepte austauschbar verwenden können und wann nicht.

Spannung und EMK sind beide formal auf die gleiche Weise definiert wie das (negative) Linienintegral des elektrischen Felds über einen bestimmten Pfad: A B E D l . Aber die Begriffe werden in unterschiedlichen Zusammenhängen verwendet. Spannung wird im elektrostatischen Kontext verwendet, wo sich keine Magnetfelder im Laufe der Zeit wesentlich ändern und das elektrische Feld durch feste elektrische Ladungen (oder chemische Potentiale usw.) erzeugt wird. EMK wird im Zusammenhang mit dem Faradayschen Gesetz verwendet, bei dem das elektrische Feld durch ein zeitlich veränderliches Magnetfeld induziert wird. In Situationen, in denen beide Quellenladungen zeitlich veränderliche Magnetfelder eine Rolle spielen, kann die Unterscheidung etwas verschwommen werden.

Ein weiterer Unterschied besteht darin, dass in einer magnetostatischen Situation der Wert des Linienintegrals wegunabhängig ist und nur von den Endpunkten abhängt. Daher wird die Spannung normalerweise als eine Eigenschaft betrachtet, die zwei Punkte im Raum in Beziehung setzt, während die EMK explizit von dem Weg abhängt, den der Draht zwischen den Endpunkten nimmt.

Aber beide haben die Wirkung, Strom durch Leiter zu drücken. Sie können EMK anstelle von Spannung in so ziemlich allen Formeln für elektrische Gleichströme durch Drähte verwenden, solange die Situation quasistatisch genug ist, dass Sie davon ausgehen können, dass das elektrische Feld entlang des Drahtes gleichmäßig ist (was in der Praxis fast immer der Fall ist Fall).

Danke, aber ich suchte nach Beweisen.
@YasirSadiq du hast eine überzeugende Erklärung, es gibt wirklich nicht viel mehr zu sagen; Wenn Sie es immer noch nicht verstehen, lesen Sie es noch einmal, dies ist das Gesetz von Faraday und es ist nicht "beweisbarer" als das Gesetz von Newton F = M A .
@YasirSadiq Es geht hauptsächlich darum, was zu klären E Und v eigentlich bedeuten. tparker hat dies bewundernswert gemacht.
@hyportnex und Philip Wood, vielen Dank. In diesem Fall ist die magnetische EMK nur im oberen Draht vorhanden, der sich im Magnetfeld befindet. Also hätten wir es tun sollen ICH = E R / 4 wenn Spannung und EMK hier ersetzt werden könnten. Aber anstatt zu haben ICH = E R / 4 wir haben ICH = E / R .
@tparker: Beachten Sie, dass das OP gefragt hat: " Warum ist der Strom in der Schleife ...? ", was eine Frage zum Strom in der Schleife ist. Ihre Antwort bezieht sich jedoch hauptsächlich auf die induzierte EMK und Spannung sowie den Unterschied zwischen diesen beiden Begriffen.

Für die kombinierte Beziehung existiert kein Beweis:

ICH = D Φ D T / R

...weil es nicht in allen Fällen zutrifft.

Dies geschieht, weil der in der Schleife induzierte Strom seinen eigenen internen Magnetfluss erzeugt, der dem externen Magnetfluss gemäß dem Lenz-Gesetz entgegenwirkt. Diese Reaktion modifiziert die Variable Φ in der obigen Gleichung.

Betrachten Sie den Grenzfall, wenn der Widerstand der kurzgeschlossenen Schleife Null ist (dies kann wirklich in supraleitenden Schleifen passieren).

Gemäß dieser Beziehung des Ohmschen Gesetzes: ICH = E / R
...irgendwelche induzierte E würde unendlich Strom in der Schleife erzeugen. Das wäre absurd.

Was in Wirklichkeit in einem solchen Fall passiert, ist, dass die SUMME des externen Flusses + des Flusses, der durch den in der Schleife induzierten Strom erzeugt wird, = KONSTANTE ist.

Sehen Sie sich diese Simulation einer R = 0-Schleife an, die sich durch das Feld eines Permanentmagneten bewegt

Die obige Simulation zeigt die Schleife, die sich über das Feld eines Magneten anstatt über ein gleichförmiges Feld bewegt, aber das Prinzip der Änderung des von der Schleife umfassten Flusses ist dasselbe.

Gegenfluss

Der Grund dafür ist, dass jede Änderung des Nettoflusses einen Wert ungleich Null erfordert E um die Schleife herum, was einen unendlichen Strom erfordert, sodass sich der Nettomagnetfluss durch die Schleife nicht ändern kann. Der Fluss von der Selbstinduktion L der Schleife muss gleich und entgegengesetzt zum externen Fluss sein, daher schließen wir, dass in einer kurzgeschlossenen Spule ohne Widerstand der maximal induzierte Strom UNABHÄNGIG ist von D Φ D T und einfach gleich ICH = Φ L .

Das ist ein großartiges Antwort- und Simulationsvideo. Wie würden Sie den letzten Satz Ihrer Antwort für den Fall ändern, dass die Schleife einen Widerstand hat (nicht supraleitend)?
Wenn die Spule nicht rein induktiv wäre und einen Widerstand hätte, müsste ich die kontinuierliche Ableitung des induzierten Stroms im Widerstand als Wärmeenergie (und die resultierende Ableitung des Gegenflusses auch) als Zeitintegral des Terms einbeziehen R ich 2 um dtund ändern Sie die Simulation so, dass sie so aussieht: youtu.be/wUaqXk6axOo . Ich würde auch schreiben, dass aufgrund dieser ohmschen Verlustleistung der maximal induzierte Strom kleiner ist als im Fall von R = 0.
Hervorragend, danke, Sir.
Wenn die Masse oder das Gewicht des Magneten viel größer ist, springt der Magnet natürlich nicht zurück - er fällt nur mit einer kleinen Verlangsamung durch die Schleife. Dasselbe kann passieren, wenn der Magnet einen kleineren Fluss hat oder die Schleife in Bezug auf den Magneten größer ist.
Welches Paket haben Sie für Ihre Simulationen verwendet?
EM-Works und C++. Unten sehen Sie eine Simulation eines schweren Magneten, der durch eine supraleitende Schleife fällt, ohne abzuprallen. youtu.be/yMg6j-UoUfo Beachten Sie, dass der Nettofluss durch die Schleife konstant bleibt und dass der Fluss vom Magneten niemals den Ring kreuzt.
Das ist fantastisch!

In dieser Situation wirkt eine magnetische Kraft auf die freien Elektronen im oberen Segment der Schleife, drückt sie nach links und erzeugt eine Potentialdifferenz vBl zwischen den beiden Enden dieses Segments. Andererseits erfährt eine geschlossene Leiterschleife, die sich vollständig innerhalb eines gleichmäßigen Magnetfelds befindet, das sich mit der Zeit ändert, eine EMK, die Elektronen um die Schleife herum drückt, aber Spannungsunterschiede werden von Punkt zu Punkt abgebaut, wenn sich der Strom durch den Widerstand der Schleife bewegt Dirigent.

Eine Potentialdifferenz, vBl, zwischen den beiden Enden dieses Segments kann erzeugt werden, wenn die Elektronen im Gleichgewicht sind, das heißt, die Nettokraft auf sie ist Null, aber hier ist das nicht der Fall.
Der Strom in der Schleife wird durch den Gesamtwiderstand der Schleife bestimmt. Ich hätte beachten sollen, dass ein Teil der Potenzialdifferenz entlang des oberen Segments durch den Widerstand dieses Segments verloren geht.
Wenn Sie schreiben " Der Strom in der Schleife wird durch den Gesamtwiderstand der Schleife bestimmt ", meinen Sie das? ICH = E / R .
Ja, und in diesem Fall ist ε = vBL
@RW Bird: Die Gleichung ICH = E / R ist in der Grenze falsch, da R gegen Null geht, weil an dieser Grenze alles ungleich Null ist E impliziert einen beliebig großen ('unendlichen') Strom. ... was absurd ist.
Echte Drähte haben einen endlichen Widerstand.
Behaupten Sie, dass supraleitende Drähte nicht real sind, ... wie in der Fiktion? Wie auch immer, wenn Ihre Gleichung am Limit versagt, dann versagt sie auch, wenn Sie sich dem Limit nähern, dh für sehr kleine Widerstände, die noch größer als Null sind. Mit anderen Worten, dies ist keine allgemeine Gleichung, die die Realität widerspiegelt, und als solche ist sie falsch. Dieser Fehler wird durch die Unkenntnis des entgegengesetzten magnetischen Flusses aufgrund des in der Schleife induzierten Stroms verursacht. Siehe physical.stackexchange.com/questions/62721/…
Sie haben Recht, ich habe die Selbstinduktivität der Schleife ignoriert.

Wenn die Schleife mit einer Geschwindigkeit nach unten geht v j , für ein Magnetfeld B, Schleifenbreite L und Teil der Schleifenhöhe im Feld = j ,

ϕ T = B ( L j ) T = L B j T = L B v j

Wie Sie sagten, wird im Leiter ein elektrisches Feld erzeugt:

E = ϕ T
Wenn die Geschwindigkeit konstant ist, E ist ebenfalls konstant, und wir haben eine statische Situation: v = E D l = R ICH .

Wenn es nicht konstant ist, erzeugt das sich ändernde Magnetfeld, das durch den Strom im Leiter erzeugt wird, ein E-Feld, das diesem Strom entgegenwirkt. Also streng genommen v R ICH in diesem Fall. In der Praxis ist dieser zweite Effekt sehr klein, abgesehen von einer großen Induktivität und einem kleinen Widerstand des Leiters.

Beide Effekte müssen zusammen ausgewertet werden, was mathematisch bedeutet, dass die 2 Maxwell-Gleichungen:

× E = B T
× B = μ 0 J + μ 0 ϵ 0 E T

müssen für alle Felder gleichzeitig erfüllt sein.

Ohmsches Gesetz in der Form

ICH = v R
Wo v Spannung zwischen zwei Punkten ist, ist lediglich ein Sonderfall des allgemeinen Ohmschen Gesetzes, das nur auf Situationen anwendbar ist, in denen alle im Metallleiter aktiven elektromotorischen Kräfte auf elektrostatische Felder zurückzuführen sind. Dies ist hier nicht der Fall, da der Strom nicht auf ein elektrostatisches Feld zurückzuführen ist, sondern auf eine spezielle elektromotorische Kraft des Leiters selbst, die auf die beweglichen Ladungen einwirkt, die den Strom bilden, die als Bewegungs-EMK bezeichnet werden . Es erscheint immer dann, wenn sich der Leiter im Magnetfeld bewegt.

Allgemeines Ohmsches Gesetz (ohne Berücksichtigung magnetischer Kräfte) ist

E + E = σ J

Wo E ist das gesamte elektrische Feld, E ist die gesamte nichtelektrische Kraft pro Ladungseinheit (in diesem Fall die Kraft aufgrund der Bewegung des Drahtes im Magnetfeld).

Wir können beide Seiten des allgemeinen Ohmschen Gesetzes für einige Segmente der Schaltung auf folgende Weise integrieren:

S T A R T   Ö F   S e N D   Ö F   S ( E + E ) D S = S T A R T   Ö F   S e N D   Ö F   S σ J D S
Die linke Seite wird als gesamte elektromotorische Kraft für das Segment bezeichnet S und wir können es bezeichnen E S .

Wenn wir uns vorstellen, dass der Schaltungsdraht ein Torus mit Querschnittsfläche ist A und Länge L , wir können die Integration über den gesamten geschlossenen Kreis durchführen, und dann erhalten wir

E C ich R C u ich T = σ A L ICH .

Wir wissen aus der High School, dass der ohmsche Widerstand des Stromkreises ist

R = σ A L
so kommen wir endlich bei der gleichung an

E C ich R C u ich T = R ICH .

Dies ist die andere (integrale) Form des allgemeinen Ohmschen Gesetzes.